A survey published in a leading medical journal in the early 1970s found that the more frequently people engaged in a...

Kanyinsola on June 29, 2019

D and C

Hi there, Would love an explanation as to why D is wrong. My initial answer was C as I can see how it is a correlation v causation argument. However, upon further reflection on the passage and choice D I went with D because it is my understanding in LSAT that you shouldn't bring in outside assumptions. Does that make sense?

Reply
Create a free account to read and take part in forum discussions.

Already have an account? log in

Ravi on June 29, 2019

@Kanyin,

Happy to help. Let's take a look at (D) and (C).

(D) says, "presumes, without providing justification, that anyone who
does not have lung disease is in good health"

It's true that the conclusion is about how exercise can help lead to a
health benefit, but the problem with (D) is that the argument in the
stimulus never makes the conclusion that anyone who doesn't have lung
disease is in good health; rather, the argument states that they
receive a benefit to their health. Thus, (D) is out.

(C) says, "concludes merely from the fact that two things are
correlated that one causes the other"

(C) is great because, as noted in the stimulus, there's a strong
correlation between exercise and a reduced risk of lung disease. The
argument then makes the conclusion that exercise is the cause of this
health benefit, which is mistaking correlation for causation. This is
just what (C) says, so it's the correct answer.

Does this make sense? Let us know if you have any other questions!